Đến nội dung

shinichigl nội dung

Có 120 mục bởi shinichigl (Tìm giới hạn từ 05-06-2020)



Sắp theo                Sắp xếp  

#523844 Đề Khảo sát chất lượng đội tuyển Chuyên Lê Thánh Tông 2014

Đã gửi bởi shinichigl on 10-09-2014 - 22:34 trong Thi HSG cấp Tỉnh, Thành phố. Olympic 30-4. Đề thi và kiểm tra đội tuyển các cấp.

$\boxed{\text{Câu 7}}$  Cho $a,b,c >0$ thỏa: $\frac{a}{\sqrt{b+c}}+\frac{b}{\sqrt{c+a}}+\frac{c}{\sqrt{a+b}} \leq \frac{\sqrt{6}}{2}$. Chứng minh $a+b+c \leq 1$

Không mất tính tổng quát, giả sử $a\geq b\geq c$ $\Rightarrow \sqrt{a+b}\geq \sqrt{a+c}\geq \sqrt{b+c}$ (1)

Ta sẽ chứng minh $\frac{b}{\sqrt{c+a}}\geq \frac{c}{\sqrt{a+b}}$

Thật vậy, ta có $\frac{b}{\sqrt{c+a}}\geq \frac{c}{\sqrt{a+b}}\Leftrightarrow b\sqrt{a+b}\geq c\sqrt{c+a}\Leftrightarrow b^{3}+b^{2}a\geq c^{3}+c^{2}a$

$\Leftrightarrow \left ( b-c \right )\left ( b^{2}+c^{2}+ab+bc+ca \right )\geq 0$ (đúng)

Tương tự, suy ra $\frac{a}{\sqrt{b+c}}\geq \frac{b}{\sqrt{c+a}}\geq \frac{c}{\sqrt{a+b}}$ (2)

Từ (1) và (2), theo bất đẳng thức Chebyshev dạng phân số ta có $\sum \frac{a}{\sqrt{b+c}}\geq 3\frac{\sum a}{\sum \sqrt{b+c}}$

$\Rightarrow 3\frac{\sum a}{\sum \sqrt{b+c}}\leq \sum \frac{a}{\sqrt{b+c}}\leq \frac{\sqrt{6}}{2}\Rightarrow \sum \sqrt{b+c}\geq \sqrt{6}\sum a$

$\Rightarrow \sum \sqrt{b+c}\geq \sqrt{6}\sum a\geq \frac{1}{\sqrt{6}}\left ( \sum \sqrt{b+c} \right )^{2}$

$\Rightarrow \sum \sqrt{b+c}\leq \sqrt{6}$

$\Rightarrow \sum a\leq \frac{\sum \sqrt{b+c}}{\sqrt{6}}\leq 1$

Dấu "=" xảy ra tại $a=b=c$




#523833 Đề Khảo sát chất lượng đội tuyển Chuyên Lê Thánh Tông 2014

Đã gửi bởi shinichigl on 10-09-2014 - 21:56 trong Thi HSG cấp Tỉnh, Thành phố. Olympic 30-4. Đề thi và kiểm tra đội tuyển các cấp.

$\boxed{\text{Câu 2}}$  Cho $a,b,c >0$ và $abc=1$. Chứng minh

$\frac{a^3}{2b^2+bc}+\frac{b^3}{2c^2+ac}+\frac{c^3}{2a^2+ab} \geq 1$

Đặt $a=\frac{x}{y};b=\frac{y}{z};c=\frac{z}{x}$ (do $abc=1$)

Bất đẳng thức được viết lại

$\sum \frac{\left (\frac{x^{2}z}{y^{2}} \right )^{2}}{2xy+z^{2}}\geq 1$

Ta có $\sum \frac{\left (\frac{x^{2}z}{y^{2}} \right )^{2}}{2xy+z^{2}}\geq \frac{\left ( \sum \frac{x^{2}z}{y^{2}} \right )^{2}}{\left ( x+y+z \right )^{2}}$ (Bất đẳng thức Cauchy - Schwarz dạng phân số)

Ta cần chứng minh $\sum \frac{x^{2}z}{y^{2}}\geq x+y+z$

Thật vậy, ta có $\frac{x^{2}z}{y^{2}}+\frac{y^{2}x}{z^{2}}+z\geq 3x$ (Bất đẳng thức AM - GM cho 3 số dương)

$\Rightarrow 2\sum \frac{x^{2}z}{y^{2}}+x+y+z\geq 3\left ( x+y+z \right )$

$\Rightarrow \sum \frac{x^{2}z}{y^{2}}\geq x+y+z$

Từ đó, ta có điều phải chứng minh

Dấu "=" xảy ra khi $x=y=z$ hay $a=b=c=1$




#523659 Chọn đội tuyển hà tĩnh 2014-2015

Đã gửi bởi shinichigl on 09-09-2014 - 19:06 trong Thi HSG cấp Tỉnh, Thành phố. Olympic 30-4. Đề thi và kiểm tra đội tuyển các cấp.

 

Câu 2: Cho dãy số $(x_n)$ được xác định bởi:
       $x_1=\frac{1}{2}; x_{n+1}=\frac{2014+x_n}{2016-x_n}$ với mọi $n=1,2,...$.
a. Chứng minh rằng dãy $(x_n)$ có giới hạn và tính giới hạn đó.
b. Với mỗi số tự nhiên $n \ge 1,$ đặt $y_n=\frac{1}{2013n+2015} \sum_{k=1}^{n} \frac{1}{x_k-2014}.$ Tính $\lim y_n$
 

a. Ta dễ dàng chứng minh bằng quy nạp $x_{n}\neq 1,\forall n=1,2,...$

$x_{n+1}-1=\frac{2\left ( x_{n}-1 \right )}{2016-x_{n}}$

$x_{n+1}-2014=\frac{2015\left ( x_{n}-2014 \right )}{2016-x_{n}}$

$\Rightarrow \frac{x_{n+1}-2014}{x_{n+1}-1}=\frac{2015}{2}\frac{x_{n}-2014}{x_{n}-1}=\left (\frac{2015}{2} \right )^{2}\frac{x_{n-1}-2014}{x_{n-1}-1}=...=\left (\frac{2015}{2} \right )^{n}\frac{x_{1}-2014}{x_{1}-1}=4027\left ( \frac{2015}{2} \right )^{n}$

$\Rightarrow x_{n+1}=\frac{2014.2^{n}-4027.2015^{n}}{2^{n}-4027.2015^{n}}$

$\Rightarrow \lim x_{n}=\lim \frac{2014.2^{n-1}-4027.2015^{n-1}}{2^{n-1}-4027.2015^{n-1}}=\lim \frac{2014.\left (\frac{2}{2015} \right )^{n-1}-4027}{\left (\frac{2}{2015} \right )^{n-1}-4027}=1$.

b. Từ công thức tìm được ở câu a) ta có

$\frac{1}{x_{n}-2014}=\frac{1}{2013.4027}\left [\left (\frac{2}{2015} \right )^{n-1}-4027 \right ]$

$\Rightarrow \sum_{k=1}^{n}\frac{1}{x_{k}-2014}=\frac{1}{2013.4027}\left [ 1+\frac{2}{2015}+\left ( \frac{2}{2015} \right )^{2}+...+\left ( \frac{2}{2015} \right )^{n-1} \right ]-\frac{n}{2013}=\frac{2015}{\left ( 2013 \right )^{2}.4027}\left [ 1-\left ( \frac{2}{2015} \right )^{n} \right ]-\frac{n}{2013}$

$\Rightarrow \lim y_{n}=\lim \left \{ \frac{2015}{\left ( 2013 \right )^{2}.4027}\left [ 1-\left ( \frac{2}{2015} \right )^{n} \right ]-\frac{n}{2013} \right \}\frac{1}{2013n+2015}=\lim \left \{ \frac{2015}{\left ( 2013 \right )^{2}.4027}\left [ 1-\left ( \frac{2}{2015} \right )^{n} \right ]\frac{1}{2013n+2015}-\frac{1}{2013^{2}+\frac{2013.2015}{n}} \right \}=-\frac{1}{2013^{2}}$.




#523648 ĐỀ THI ĐỀ NGHỊ OLYMPIC 30/4 NĂM 2014 TRƯỜNG THPT CHUYÊN HÙNG VƯƠNG - GIA LAI

Đã gửi bởi shinichigl on 09-09-2014 - 17:38 trong Thi HSG cấp Tỉnh, Thành phố. Olympic 30-4. Đề thi và kiểm tra đội tuyển các cấp.

Áp dụng bất đẳng thức AM-GM ta có 

$\sum \frac{x}{x^{2}+1}=\sum \frac{x}{x^{2}+\frac{1}{9}+\frac{8}{9}}\leq \sum\frac{x}{\frac{2x}{3}+\frac{8}{9}}\leq \sum \frac{x}{25}(\frac{1}{\frac{2x}{3}}+\frac{4}{\frac{2}{9}})=\sum \frac{x}{25}.\frac{3}{2x}+18.\frac{x}{25}=\frac{9}{50}+\frac{18\sum x}{25}=\frac{9}{50}+\frac{18}{25}=\frac{9}{10}$

Dấu bằng xảy ra khi $a=b=c=\frac{1}{3}$

A-Q:)

"x,y,z" là số thực mà bạn




#523603 ĐỀ THI ĐỀ NGHỊ OLYMPIC 30/4 NĂM 2014 TRƯỜNG THPT CHUYÊN HÙNG VƯƠNG - GIA LAI

Đã gửi bởi shinichigl on 09-09-2014 - 11:46 trong Thi HSG cấp Tỉnh, Thành phố. Olympic 30-4. Đề thi và kiểm tra đội tuyển các cấp.

Câu 1.

Giải phương trình $x\sqrt{x^{2}+6}+\left ( x+1 \right )\sqrt{x^{2}+2x+7}=\frac{13}{5}\left ( 2x+1 \right )$

 

Câu 2.

Cho đường tròn $\left ( \omega \right )$. Trên $\left ( \omega \right )$, lấy hai điểm cố định $A$ và $B$ sao cho các tiếp tuyến tại $A$ và $B$ của $\left ( \omega \right )$ cắt nhau. Gọi $P$ là giao điểm của hai tiếp tuyến đó. Một điểm $C$ di động trên cung lớn $AB$ của $\left ( \omega \right )$. Đường thẳng $CP$ cắt lại đường tròn $\left ( \omega \right )$ tại điểm $D$ (khác $C$). Gọi $I$ và $J$ lần lượt là trung điểm của $CD$ và $AB$. Hãy xác định vị trí của điểm $C$ trên cung lớn $AB$ của đường tròn $\left ( \omega \right )$ để biểu thức $S=JC+JD-IA-IB+CD$ đạt giá trị lớn nhất.

 

Câu 3.

Cho các số thực $x,y,z$ sao cho $x+y+z=1$. Chứng minh rằng

$\frac{x}{x^{2}+1}+\frac{y}{y^{2}+1}+\frac{z}{z^{2}+1}\leq \frac{9}{10}$

 

Câu 4.

Tìm tất cả các cặp số nguyên dương $\left ( x,y \right )$ sao cho $\frac{x^{2}+y^{2}}{x-y}$ là số nguyên và là ước của $1995$.

 

Câu 5.

Với mỗi $n\in \mathbb{N}^{*}$, một $n$ - cầu thang là một hình gồm $\frac{n\left ( n+1 \right )}{2}$ ô vuông đơn vị (gọi tắt là ô): $1$ ô ở hàng thứ nhất, $2$ ô liên tiếp nhau ở hàng thứ hai,..., $n$ ô liên tiếp nhau ở hàng thứ $n$ (số thứ tự của các hàng được tính theo chiều từ trên xuống dưới) sao cho tất cả các ô tận cùng bên trái của mỗi hàng thì nằm trên cùng một cột. Gọi $f\left ( n \right )$ là số bé nhất các hình vuông với cạnh nguyên dương cần dùng để phủ khít một $n$ - cầu thang (các hình vuông không nhất thiết bằng nhau nhưng phải không được chờm lên nhau hay chờm ra ngoài $n$ - cầu thang). Chẳng hạn, $f\left ( 1 \right )=1,f\left ( 2 \right )=3,f\left ( 4 \right )=7$. Hãy tìm tất cả các số $n\in \mathbb{N}^{*}$ để $f\left ( n \right )=n$.

 

Câu 6.

Cho $f:\mathbb{N}^{*}\rightarrow \mathbb{N}$ thoã mãn các tính chất:

1. $\forall m,n\in \mathbb{N}^{*}$: $f\left ( m+n \right )-f\left ( m \right )-f\left ( n \right )$ lấy giá trị $0$ hoặc $1$.

2. $f\left ( 2 \right )=0$ và $f\left ( 3 \right )>0$.

3. $f\left ( 9999 \right )=3333$.

Tính $f\left ( 2014 \right )$.




#523349 ĐỀ THI CHỌN ĐỘI TUYỂN HSG QUỐC GIA TỈNH QUẢNG TRỊ năm 2012

Đã gửi bởi shinichigl on 07-09-2014 - 20:24 trong Thi HSG cấp Tỉnh, Thành phố. Olympic 30-4. Đề thi và kiểm tra đội tuyển các cấp.

KÌ THI CHỌN ĐỘI TUYỂN HSG QUỐC GIA

Khóa ngày 18 tháng 9 năm 2012

MÔN TOÁN (Vòng II)

Thời gian làm bài: 180 phút

 

Câu 1. (4,0 điểm)

Giải hệ phương trình: $\left\{\begin{matrix} 2y=\left ( x-y \right )\left ( y+3x \right )                    (1) & \\ 3\frac{y^{2}}{x^{2}}+2\frac{y^{2}}{x}+x-3y=0     (2) & \end{matrix}\right.$

 

Câu 2. (4,0 điểm)

Cho tam giác ABC có $\widehat{BAC}=120^{o}$. Chứng minh rằng khoảng cách từ tâm đường tròn ngoại tiếp đến trực tâm của tam giác ABC bằng $AB+AC$.

 

Câu 3. (4,0 điểm)

Cho các số dương $x_{1},x_{2},...,x_{n}$, nằm trên một đoạn $\Delta$ có độ dài bằng $2$, với $n\geq 2$. Chứng minh rằng:

$x_{1}+x_{2}+...+x_{n}\leq \sqrt{x_{1}x_{2}+1}+\sqrt{x_{2}x_{3}+1}+...+\sqrt{x_{n}x_{1}+1}\leq x_{1}+x_{2}+...+x_{n}+n$.

 

Câu 4. (4,0 điểm)

Cho các dãy số $\left ( a_{n} \right )$ và $\left ( b_{n} \right )$ thõa mãn các điều kiện: $a_{1}=1,b_{1}=2$ thì

$a_{n+1}=\frac{1+a_{n}+a_{n}b_{n}}{b_{n}},b_{n+1}=\frac{1+b_{n}+a_{n}b_{n}}{a_{n}}$

Tính $\lim_{n\rightarrow \infty }\frac{a_{n}}{\sqrt{n}}$.

 

Câu 5. (4,0 điểm)

Có bao nhiêu số tự nhiên có $2013$ chữ số mà số các chữ số $0$ xuất hiện là chẵn?

 

P\s: Mình không chắc đề Câu 4 lắm




#523328 Topic về tổ hợp, các bài toán về tổ hợp

Đã gửi bởi shinichigl on 07-09-2014 - 19:15 trong Tổ hợp và rời rạc

Bài 45 (Thuật toán) Cho hai đống đá, một đống có $a$ hòn đá, đống kia có $b$ hòn đá. Hai người chơi, mỗi người đến lượt mình được lấy một hòn đá từ một trong hai đống, hoặc lấy phần nguyên của một nửa số hòn đá từ đống thứ nhất, hoặc lấy phần nguyên một nửa số hòn đá từ đống thứ nhất, hoặc lấy phần nguyên một nửa số hòn đá từ đống thứ hai, hoặc lấy từ hai đống số hòn đá như nhau. Người lấy được hòn đá cuối cùng là người chiến thắng. Hãy tìm tất cả các cặp $(a,b)$ trong đó $a\leq 8,b\leq 13$ sao cho người đi sau có chiến thuật thắng.




#523327 Topic về tổ hợp, các bài toán về tổ hợp

Đã gửi bởi shinichigl on 07-09-2014 - 19:06 trong Tổ hợp và rời rạc

Bài 44 (Thuật toán) Có hai đống đá có $n$ hòn và đống kia có $k$ hòn. Cứ mỗi phút một máy tự động lại chọn một đống có số hòn đá là chẵn và chuyển một nửa số hòn đá của đống đá được chọn sang đống kia. Nếu cả hai đống đều có số hòn đá là chẵn thì máy sẽ chọn ngẫu nhiên một đống. Nếu trong hai đống số hòn đá đều là lẻ thì máy sẽ ngừng làm việc. Hỏi tồn tại bao nhiêu cặp sắp thứ tự $(n,k)$, với $n$ và $k$ là các số tự nhiên không vượt quá $2013$, để máy tự động sau một khoảng thời gian hữu hạn sẽ dừng?




#522755 Chứng minh rằng ta có thể đi lại bằng máy bay giữa hai thành phố bất kì nhưng...

Đã gửi bởi shinichigl on 04-09-2014 - 17:00 trong Tổ hợp và rời rạc

Cần góp ý cho lời giải của mình:Ta có $2014=21.93+61$ nên chứng tỏ tồn tại $21$ nhóm gồm $93$ thành phố có đường nối với tất cả thành phố còn lại và dư ra $61$ nhóm thành phố có không có đường nối với tất cả các thành phố còn lại. Do luôn tồn tại đường đi giữa $2$ thành phố nên trong trường hợp xấu nhất ta phải qua $61$ thành phố đó và đến với $1$ nhóm $93$ thành phố kia,khi đó ta cần $1$ lần bay nữa. Vậy tối đa có $62$ chuyến bay $Q.E.D$

A-Q:)

Sao suy ra cái này được vậy bạn "tồn tại $21$ nhóm gồm $93$ thành phố có đường nối với tất cả thành phố còn lại và dư ra $61$ nhóm thành phố có không có đường nối với tất cả các thành phố còn lại"




#522357 A={1;2;3...2000}.. tính số tập con B của A mà tốn tại x,y thuộc A s...

Đã gửi bởi shinichigl on 02-09-2014 - 08:27 trong Tổ hợp và rời rạc

tại sao "có $n$ cặp như vậy nên có $3^{n}$ "tập hà bạn 

Ta phân hoạch tập $A$ thành $n$ cặp dạng $(a,2n+1-a)$

Để lập một tập con $X$ của $A$ ta có các cách sau:

i) đặt $a$ vào $X$ nhưng không đặt $2n+1-a$ vào $X$

ii) đặt $2n+1-a$ vào $X$ nhưng không đặt $a$ vào $X$

iii) đặt $a$ vào $X$ và đặt $2n+1-a$ vào $X$

iiii) không đặt $a$ vào $X$ và không đặt $2n+1-a$ vào $X$

Ta xét tập $B$. Do đề bài yêu cầu không tồn tại hai phần tử $x,y$ thuộc tập $B$ sao cho $x+y=2n+1$ nên ta bỏ đi trường hợp iii)

Vậy từ mỗi cặp $(a,2n+1-a)$ ta có 3 cách để thành lập tập $B$, mà có $n$ cặp như vậy nên ta có $3^{n}$ tập con $B$




#522176 Giải phương trình $x^5+10x^3+20x-18=0$

Đã gửi bởi shinichigl on 31-08-2014 - 23:41 trong Phương trình - Hệ phương trình - Bất phương trình

Giải phương trình $$x^5+10x^3+20x-18=0$$

Ta có công thức sau $\frac{1}{2}\left ( a^{5}-\frac{1}{a^{5}} \right )=16m^{5}+20m^{3}+5m$, trong đó $m=\frac{1}{2}\left ( a-\frac{1}{a} \right )$ (đa thức Chebyshev) (1)

Ta đặt $x=\sqrt{2}\left ( a-\frac{1}{a} \right )$ ($a\neq 0$)

Từ đó, phương trình được viết lại 

$4\sqrt{2}\left ( a-\frac{1}{a} \right )^{5}+20\sqrt{2}\left ( a-\frac{1}{a} \right )^{3}+20\sqrt{2}\left ( a-\frac{1}{a} \right )-18=0$

$\Leftrightarrow \frac{1}{2}\left ( a-\frac{1}{a} \right )^{5}+\frac{5}{2}\left ( a-\frac{1}{a} \right )^{3}+\frac{5}{2}\left ( a-\frac{1}{a} \right )=\frac{9\sqrt{2}}{8}$ (2)

Áp dụng công thức (1) vào phương trình (2) ta được

$\frac{1}{2}\left ( a^{5}-\frac{1}{a^{5}} \right )=\frac{9\sqrt{2}}{8}\Leftrightarrow \left ( a^{5} \right )^{2}-\frac{9\sqrt{2}}{4}a^{5}-1=0\Leftrightarrow a=\sqrt[5]{\frac{9\pm \sqrt{113}}{4\sqrt{2}}}$

Phương trình có nghiệm duy nhất $x=\sqrt{2}\left ( \sqrt[5]{\frac{9 + \sqrt{113}}{4\sqrt{2}}}-\sqrt[5]{\frac{4\sqrt{2}}{9 + \sqrt{113}}} \right )$




#522174 chứng minh quy tắc nhân của giới hạn

Đã gửi bởi shinichigl on 31-08-2014 - 23:12 trong Dãy số - Giới hạn

Cho hai dãy số: $x_{n}$ có giới hạn hữu hạn là $a$ và $y_{n}$ có giới hạn hữu hạn là $b$. Chứng minh $\lim_{n\rightarrow +\infty }\left ( x_{n} .y_{n}\right )=a.b$




#522171 x,y,z$\geq 0 ,yz=1+zx+zy$

Đã gửi bởi shinichigl on 31-08-2014 - 22:50 trong Bất đẳng thức - Cực trị

x,y,z$\geq 0 ,xy=1+zx+zy$

TÌm Max của $P=\frac{x^{2}}{x^{2}+1}+\frac{y^{2}}{y^{2}+1}+\frac{z}{z^{2}+1}$

Đặt $x=tan\alpha $, $y=tan\beta$ $\Rightarrow z=-\frac{1}{tan\left ( \alpha +\beta  \right )}$

Từ đó, bất đẳng thức được viết lại $sin^{2}\alpha + sin^{2}\beta - cos\left (\alpha +\beta   \right )sin\left (\alpha +\beta  \right ) $




#521983 chứng minh tồn tại số nguyên dương k

Đã gửi bởi shinichigl on 30-08-2014 - 18:50 trong Tổ hợp và rời rạc

Trong 1 cuộc hội thảo khoa học mỗi người tham dự đều quen ít nhất 3 người khác. Chứng minh rằng tồn tại 1 số nguyên dương $k$ không chia hết cho 3 sao cho ta có thể chọn ra được $k$ người và xếp ngồi quanh 1 bàn tròn sao cho mỗi người đều ngồi giữa 2 người quen nhau




#521980 Chứng minh rằng ta có thể đi lại bằng máy bay giữa hai thành phố bất kì nhưng...

Đã gửi bởi shinichigl on 30-08-2014 - 18:44 trong Tổ hợp và rời rạc

Giả sử có 2014 thành phố mà từ mỗi thành phố đều có đường bay trực tiếp đến ít nhất 93 thành phố khác và luôn có thể đi từ thành phố bất kì này đến 1 thành phố bất kì khác bằng máy bay. Chứng minh rằng ta có thể đi lại bằng máy bay giữa hai thành phố bất kì nhưng không quá 62 lần chuyển máy bay




#521977 $2f(x)+f(y)=3f(2x+y)$

Đã gửi bởi shinichigl on 30-08-2014 - 18:08 trong Phương trình hàm

 

2) Cho $f$ $\mathbb{R}\rightarrow \mathbb{R}$ thỏa $f(x+y)=f(x)+f(y)$    (1)

a. CMR: $f(rx)=rf(x),\forall x\epsilon \Re ,r\epsilon \mathbb{Q}$
b. CMR: Giả sử $f$ liên tục thì $f(x)=xf(1),\forall x\epsilon \mathbb{R}$    

a. Trong (1) lấy $y=x$ ta được $f\left ( 2x \right )=2f\left ( x \right )$, $\forall x\in \mathbb{R}$ (2)

Trong (2) lấy $x=0$ ta được $f(0)=0$.

Từ (1) và (2) và bằng phương pháp quy nạp ta chứng minh được $f\left ( nx \right )=nf\left ( x \right )$, $\forall x\in \mathbb{R},\forall n\in \mathbb{N}$ (3)

Trong (1) lấy $y=-x$ và sử dụng $f(0)=0$ ta được $f\left ( -x \right )=-f\left ( x \right )$, $\forall x\in \mathbb{R}$ (4)

Bởi vậy khi $n=-1,-2,...$, sử dụng (3) và (4) ta có $f\left ( nx \right )=f\left ( -n\left ( -x \right ) \right )=-nf\left ( -x \right )=nf\left ( x \right )$, $\forall x\in \mathbb{R}$ (5)

Từ (3) và (5) suy ra $f\left ( nx \right )=nf\left ( x \right )$, $\forall x\in \mathbb{R},\forall n\in \mathbb{Z}$ (6)

Với mọi $m,n\in \mathbb{Z}$ và $m>0$, sử dụng (6) ta có $f\left ( nx \right )=f\left ( m.\frac{n}{m}.x \right )=mf\left ( \frac{n}{m}.x \right )=nf\left ( x \right )$, $\forall x\in \mathbb{R}$

Suy ra, với mọi $m,n\in \mathbb{Z}$ và $m>0$, ta có $f\left ( \frac{n}{m}.x \right )=\frac{n}{m}f\left ( x \right )$, $\forall x\in \mathbb{R}$

Bởi vậy $f\left ( rx \right )=rf\left ( x \right )$, $\forall x\in \mathbb{R},\forall r\in \mathbb{Q}$ (8)

 

b. Trong (8) lấy $x=1$ ta được $f\left ( r \right )=rf\left ( 1 \right )$, $\forall r\in \mathbb{Q}$

Với mỗi $x\in \mathbb{R}$ tồn tại dãy số hữu tỉ $\left \{ r_{n} \right \}_{n=1}^{+\infty }$ sao cho $\lim_{n\rightarrow +\infty }r_{n}=x$.

Vì $f$ liên tục nên

$f\left ( x \right )=f\left ( \lim_{n\rightarrow +\infty }r_{n} \right )=\lim_{n\rightarrow +\infty }f\left ( r_{n} \right )=\lim_{n\rightarrow +\infty }r_{n}f\left ( 1 \right )=f\left ( 1 \right )\lim_{n\rightarrow +\infty }r_{n}=f\left ( 1 \right )x$

Vậy $f\left ( x \right )=xf\left ( 1 \right )$, $\forall x\in \mathbb{R}$




#521912 A={1;2;3...2000}.. tính số tập con B của A mà tốn tại x,y thuộc A s...

Đã gửi bởi shinichigl on 30-08-2014 - 00:06 trong Tổ hợp và rời rạc

Sorry nếu nãy bạn nào đọc bài rồi nhưng mình nghĩ đây là $2001$  

Ta giải bài toán tổng quát là tập $A$ là tập $2n$ số tự nhiên đầu tiên và tồn tại $x+y=2n+1$

Số tập con của $A$ là $2^{2n}$

Ta đếm số tập con không tồn tại $x+y=2n+1$

Xét các cặp $(a,2n+1-a)$ đặt vào các tập trong các tập con của $A$ , xét tập $X$

+ Đặt $a$ vào $X$ thì không đặt $2n+1-a$ vào $X$

+ Đặt $2n+1-a$ vào $X$ thì không đặt $a$ vào $X$

+ Cả hai không đặt vào $X$

Rõ ràng có $n$ cặp như vậy nên có $3^{n}$ tập

Do vậy có $2^{2n}-3^{n}$ tập 

Mình nghĩ đáp số phải là $3^{n}$ chứ




#521126 Chứng minh rằng tồn tại hai hình có diện tích phần chung không nhỏ hơn 3/20

Đã gửi bởi shinichigl on 24-08-2014 - 22:17 trong Tổ hợp và rời rạc

Trong hình chữ nhật diện tích 1 có 5 hình có diện tích mỗi hình bằng 1/2

a) Chứng minh rằng tồn tại hai hình có diện tích phần chung không nhỏ hơn 3/20

b) Chứng minh rằng tồn tại hai hình có diện tích phần chung không nhỏ hơn 1/5

c) Chứng minh rằng tồn tại ba hình có diện tích phần chung không nhỏ hơn 1/20




#521117 Chứng minh tồn tại hai ủy ban có chung ít nhất 4 thành viên

Đã gửi bởi shinichigl on 24-08-2014 - 21:44 trong Tổ hợp và rời rạc

(Olympic toàn nước Nga, 1996) Trong Duma có 1600 đại biểu, tạo thành 16000 ủy ban, mỗi ủy ban có 80 người. Chứng minh rằng tồn tại hai ủy ban có chung ít nhất 4 thành viên




#511139 Chứng minh rằng đường thẳng Euler của tam giác ABC đi qua D

Đã gửi bởi shinichigl on 06-07-2014 - 09:12 trong Hình học

http://forum.mathsco...ead.php?t=46791
bạn xem tại đây,bài 9 bài 10 gì đó :D

Bạn giải luôn đi, chứ mình kiếm không thấy




#510898 Chứng minh rằng đường thẳng Euler của tam giác ABC đi qua D

Đã gửi bởi shinichigl on 05-07-2014 - 11:00 trong Hình học

Cho tam giác ABC với góc A không vuông. Gọi D là một điểm sao cho $\angle DBA=\angle BAC=\angle DCA$. Chứng minh rằng đường thẳng Euler của tam giác ABC đi qua D.




#504119 $\left\{\begin{matrix} f(n+1)> f(n)...

Đã gửi bởi shinichigl on 05-06-2014 - 08:57 trong Phương trình hàm

Từ giả thiết $f(n)<f(n+1)$, $\forall n\in \mathbb{N}$, ta suy ra được $f(x)<f(y)$, $\forall x<y$ ($x,y\in \mathbb{N}$)

Cũng từ giả thiết $f(n)<f(n+1)$, $\forall n\in \mathbb{N}$, ta suy ra $f(n)+1\leq f(n+1)$, $\forall n\in \mathbb{N}$

Ta giả sử $f(n)+1< f(n+1)$, $\forall n\in \mathbb{N}$ (1)

Thay $n$ bởi $f(n)$ vào (1) ta có $f(f(n))+1< f(f(n)+1)<f(f(n+1))$, $\forall n\in \mathbb{N}$ (2)

Mặt khác

Thay $n$ bởi $n+1$ vào giả thiết $f(f(n))=n+2004$, $\forall n\in \mathbb{N}$ ta có $f(f(n+1))=n+2005$, $\forall n\in \mathbb{N}$

Suy ra $f(f(n+1))-f(f(n))=1$, $\forall n\in \mathbb{N}$ hay $f(f(n+1))=f(f(n))+1$, $\forall n\in \mathbb{N}$ (3)

Từ (2) và (3) ta thấy mâu thuẫn, suy ra $f(n)+1=f(n+1)$, $\forall n\in \mathbb{N}$ (4)

Từ (4) bằng chứng minh quy nạp ta dễ dàng thu được $f(n)=n+f(0)$, $\forall n\in \mathbb{N}$ (5)

Thay (5) vào giả thiết $f(f(n))=n+2004$, $\forall n\in \mathbb{N}$, ta tìm được $f(n)=n+1002$, $\forall n\in \mathbb{N}$

Vậy hàm số cần tìm là $f(n)=n+1002$, $\forall n\in \mathbb{N}$




#501869 $f(1)=2, f(f(n))=f(n)+n, f(n)<f(n+1), \forall n \in \...

Đã gửi bởi shinichigl on 26-05-2014 - 23:27 trong Phương trình hàm

Ngoài ra, mình cũng tìm ra được một hàm thoã mãn đề bài nữa là $f(n)=\left [ \frac{\sqrt{5}+1}{2}n+\frac{1}{2} \right ]$, $\forall n\in \mathbb{N}$




#501074 $f(1)=2, f(f(n))=f(n)+n, f(n)<f(n+1), \forall n \in \...

Đã gửi bởi shinichigl on 23-05-2014 - 22:13 trong Phương trình hàm

Đặt $a=\frac{\sqrt{5}-1}{2}$ và $g(n)=n+\left [ a(n+1) \right ]$, $\forall n\in \mathbb{N}$

Ta có $g(n)\in \mathbb{N}$, $\forall n\in \mathbb{N}$

Thay $n=1$, ta có $g(1)=2$, thoã mãn điều kiện $f(1)=2$

Tiếp theo, ta chứng minh $g(g(n))=g(n)+n$, $\forall n\in \mathbb{N}$

Thật vậy,

$g\left ( g\left ( n \right ) \right )=n+\left [ a\left ( n+1 \right ) \right ]+\left [ a\left ( n+\left [ a\left ( n+1 \right ) \right ]+1 \right ) \right ]=n+\left [ a\left ( n+1 \right ) \right ]+\left [ a\left ( n+1 \right )+a\left [ a\left ( n+1 \right ) \right ] \right ]$, $\forall n\in \mathbb{N}$

Mặt khác,

$\left [ a(n+1)+a\left [ a(n+1) \right ] \right ]<a(n+1)+a^2(n+1)=n+1$, $\forall n\in \mathbb{N}$

$\left [ a\left ( n+1 \right )+a\left [ a\left ( n+1 \right ) \right ] \right ]>a\left ( n+1 \right )+a^{2}\left ( n+1 \right )-a-1=n-a>n-1$, $\forall n\in \mathbb{N}$

Suy ra $n-1<\left [ a(n+1)+a\left [ a(n+1) \right ] \right ]<n+1$, $\forall n\in \mathbb{N}$

Suy ra $\left [ a(n+1)+a\left [ a(n+1) \right ] \right ]=n$, $\forall n\in \mathbb{N}$

Suy ra $g(g(n))=n+\left [ a(n+1) \right ]+\left [ a(n+1)+a\left [ a(n+1) \right ] \right ]=n+\left [ a(n+1) \right ]+n=g(n)+n$, $\forall n\in \mathbb{N}$,

thoã mãn $f(f(n))=f(n)+n$, $\forall n\in \mathbb{N}$

Cuối cùng, ta chứng minh $g(n)<g(n+1)$, $\forall n\in \mathbb{N}$

Thật vậy,

ta có $n+\left [ a(n+1) \right ]<n+1+\left [ a(n+2) \right ]$, $\forall n\in \mathbb{N}$

hay $g(n)<g(n+1)$, $\forall n\in \mathbb{N}$,

thoã mãn $f(n)<f(n+1)$, $\forall n\in \mathbb{N}$,

Từ 3 khẳng định trên, ta chứng minh được hàm số $g(n)$ thoã mãn đề bài,

Vậy tồn tại hàm số $f$ thoã mãn đề bài, và ta cũng chỉ ra rằng một trong các hàm đó là $f(n)=n+\left [ \frac{\sqrt{5}-1}{2}(n+1) \right ]$, $\forall n\in \mathbb{N}$.




#500411 $f\left ( f\left ( x \right )+y \right )=x+f\le...

Đã gửi bởi shinichigl on 20-05-2014 - 22:58 trong Phương trình hàm

Tìm các hàm $f:Q\rightarrow R$ thỏa:

           $f\left ( f\left ( x \right )+y \right )=x+f\left ( y \right ),\forall x,y\in Q$

Mình đưa ra lời giải nhưng chưa biết đúng hay sai, mong các bạn nhận xét

Cho $x=y=0$: $f(f(0))=f(0)$  $\Rightarrow f(0)\in \mathbb{Q}$ (do $f:\mathbb{Q}\rightarrow \mathbb{R}$ )

Đặt $f(0)=a$, ta có: $f(a)=a \Rightarrow f(f(a))=a$

Cho $x=a$, $y=0$: $f(f(a))=2a$

Từ đó ta suy ra $a=0$ hay $f(0)=0$

Cho $y=0$: $f(f(x))=x$, $\forall x\in \mathbb{Q}$  $\Rightarrow f(x)\in \mathbb{Q}$ (do $f:\mathbb{Q}\rightarrow \mathbb{R}$ )

Từ đây ta cũng suy ra được hàm $f$ là một hàm đơn ánh

Thay $f(y)$ bởi $y$: $f(f(x)+f(y))=x+y$,  $\forall x,y\in \mathbb{Q}$

Mà ta lại có: $f(f(x+y))=x+y$, $\forall x,y\in \mathbb{Q}$

Suy ra: $f(f(x)+f(y))=f(f(x+y))$  $\Rightarrow f(x)+f(y)=f(x+y)$ (do $f$ đơn ánh),  $\forall x,y\in \mathbb{Q}$

Dễ thấy $f$ cộng tính nên $f(x)=kx$ ($k\in \mathbb{Q}$), $\forall x\in \mathbb{Q}$ $\Rightarrow f(f(x))=k^2x$, $\forall x\in \mathbb{Q}$ $\Rightarrow k=\pm 1$

Thử lại, ta được hai hàm số cấn tìm là $f(x)=x$, $\forall x\in \mathbb{Q}$ hoặc $f(x)=-x$, $\forall x\in \mathbb{Q}$